Skip to main content
Log in

Dyson Ferrari–Spohn diffusions and ordered walks under area tilts

  • Published:
Probability Theory and Related Fields Aims and scope Submit manuscript

Abstract

We consider families of non-colliding random walks above a hard wall, which are subject to a self-potential of tilted area type. We view such ensembles as effective models for the level lines of a class of \(2+1\)-dimensional discrete-height random surfaces in statistical mechanics. We prove that, under rather general assumptions on the step distribution and on the self-potential, such walks converge, under appropriate rescaling, to non-intersecting Ferrari–Spohn diffusions associated with limiting Sturm–Liouville operators. In particular, the limiting invariant measures are given by the squares of the corresponding Slater determinants.

This is a preview of subscription content, log in via an institution to check access.

Access this article

Price excludes VAT (USA)
Tax calculation will be finalised during checkout.

Instant access to the full article PDF.

Fig. 1
Fig. 2
Fig. 3

Similar content being viewed by others

References

  1. Billingsley, P.: Convergence of Probability Measures. Wiley, New York (1968)

    MATH  Google Scholar 

  2. Bornemann, F.: On the scaling limits of determinantal point processes with kernels induced by Sturm–Liouville operators. Preprint. arXiv:1104.0153 (2011)

  3. Caputo, P., Lubetzky, E., Martinelli, F., Sly, A., Toninelli, F.L.: Scaling limit and cube-root fluctuations in SOS surfaces above a wall. Preprint. arXiv:1302.6941 (2013)

  4. Caputo, P., Martinelli, F., Toninelli, F.L.: On the probability of staying above a wall for the (2+1)-dimensional SOS model at low temperature. Preprint. arXiv:1406.1206 (2014)

  5. Coddington, E.A., Levinson, N.: Theory of Ordinary Differential Equations. McGraw-Hill Book Company Inc, New York (1955)

    MATH  Google Scholar 

  6. Denisov, D., Wachtel, V.: Random walks in cones. Ann. Probab. 43(3), 992–1044 (2015)

    Article  MathSciNet  MATH  Google Scholar 

  7. Duits, M.: On global fluctuations for non-colliding processes. Preprint. arXiv:1510.08248 (2014)

  8. Duraj, J., Wachtel, V.: Invariance principles for random walks in cones. Preprint. arXiv:1508.07966 (2015)

  9. Esseen, C.G.: On the concentration function of a sum of independent random variables. Z. Wahrscheinlichkeitstheorie und Verw. Gebiete 9, 290–308 (1968)

    Article  MathSciNet  MATH  Google Scholar 

  10. Ferrari, P.L., Spohn, H.: Constrained Brownian motion: fluctuations away from circular and parabolic barriers. Ann. Probab. 33(4), 1302–1325 (2005)

  11. Garbit, R.: Brownian motion conditioned to stay in a cone. J. Math. Kyoto Univ. 49, 573–592 (2009)

    Article  MathSciNet  MATH  Google Scholar 

  12. Hryniv, O., Velenik, Y.: Some rigorous results on semiflexible polymers. I. Free and confined polymers. Stoch. Process. Appl. 119(10), 3081–3100 (2009)

  13. Ioffe, D., Shlosman, S.: Ising model fog drip: the first two droplets. In: In and Out of Equilibrium. 2, Progress in Probability, vol. 60, Birkhäuser, Basel, pp. 365–381 (2008)

  14. Ioffe, D., Shlosman, S., Toninelli, F.L.: Interaction versus entropic repulsion for low temperature Ising polymers. J. Stat. Phys. 158(5), 1007–1050 (2015)

    Article  MathSciNet  MATH  Google Scholar 

  15. Ioffe, D., Shlosman, S., Velenik, Y.: An invariance principle to Ferrari–Spohn diffusions. Commun. Math. Phys. 336(2), 905–932 (2015)

    Article  MathSciNet  MATH  Google Scholar 

  16. Ioffe, D., Velenik, Y.: Ballistic phase of self-interacting random walks. In: Mörters, P., Penrose, M., Schwetlick, H., Zimmer, J. (eds.) Analysis and Stochastics of Growth Processes and Interface Models, pp. 55–79. Oxford University Press, Oxford (2008)

  17. Karlin, S., McGregor, J.: Coincidence probabilities. Pac. J. Math. 9, 1141–1164 (1959)

    Article  MathSciNet  MATH  Google Scholar 

  18. König, W., Schmid, P.: Brownian motion in a truncated Weyl chamber. Markov Process. Relat. Fields 17(4), 499–522 (2011)

    MathSciNet  MATH  Google Scholar 

  19. Soshnikov, A.: Determinantal random point fields. Russ. Math. Surveys 55(5), 923–975 (2000)

    Article  MathSciNet  MATH  Google Scholar 

  20. Uchiyama, K.: One dimensional lattice random walks with absorption at a point/on a half line. J. Math. Soc. Jpn. 63(2), 675–713 (2011)

    Article  MathSciNet  MATH  Google Scholar 

  21. Velenik, Y.: Entropic repulsion of an interface in an external field. Probab. Theory Relat. Fields 129(1), 83–112 (2004)

    Article  MathSciNet  MATH  Google Scholar 

Download references

Acknowledgements

Dmitry Ioffe was supported by the Israeli Science Foundation Grant 1723/14. Yvan Velenik was partially supported by the Swiss National Science Foundation.

Author information

Authors and Affiliations

Authors

Corresponding author

Correspondence to Dmitry Ioffe.

Appendices

Appendix 1: Strong approximation techniques

In order to prove (I.1), we are going to apply strong approximation techniques from [6]. By rescaling, it is sufficient to consider the case \(t=1\).

In the sequel, \(\hat{\mathbf {P}}^{\underline{r}}_+\) denotes the restriction of the law of the n-dimensional Brownian motion \(\underline{B}\) started at \(\underline{r}\) to the set \(\mathbb {A}^+_{n}\).

Define

$$\begin{aligned} O_\epsilon (\underline{z}) = \left\{ \underline{y}\,:\, |y_i-z_i| \le { \frac{\epsilon }{3} } \text { for all } i\right\} . \end{aligned}$$

It follows easily from [6, Lemma 17] that

uniformly in \(\underline{r},\underline{z}\in \mathbb {A}_{n, \lambda }^{+, {\mathsf r}}\). This implies that there exists a constant \(c(\epsilon ,\eta ,\gamma )>0\) such that

$$\begin{aligned} \hat{\mathbf {P}}^{\underline{r}}_{+,\lambda } (\underline{x}^\lambda (1-\gamma ) \in O_\epsilon (\underline{z}) , \max _{t\le 1-\gamma } x^\lambda _n(t) \le 2\eta ) \ge c(\epsilon ,\eta ,\gamma ) , \end{aligned}$$
(7.31)

for all \(\underline{r},\underline{z}\in \mathbb {A}_{n, \lambda }^{+, {\mathsf r}}\). Since \(O_\epsilon (\underline{z})\) is separated from the boundary of \(\mathbb {A}_n^+\), we may choose \(\gamma \) so small that the probability that the random walk \(x^\lambda \) started at \(\underline{y}\in O_\epsilon (\underline{z})\) has, at time \(\gamma \), the value \(\underline{z}\) and leaves \(\mathbb {A}_{n,\lambda }^+\) before time \(\gamma \) is quite small. This heuristic is made precise in [6, Lemma 29]. In our notation, we can state that result as follows: There exist \(a>0\) and \(c_1<\infty \) such that

$$\begin{aligned}&\hat{\mathbf {P}}^{\underline{y}}_{+,\lambda } ( \underline{x}^\lambda (\gamma ) = \underline{z}, \max _{t\le \gamma } x^\lambda _n(t) \le 2\eta ) \\&\quad \ge \hat{\mathbf {P}}^{\underline{y}}_\lambda ( \underline{x}^\lambda (\gamma ) = \underline{z}, \max _{t\le \gamma } x^\lambda _n(t) \le 2\eta ) - c_1 \gamma ^{-n/2} e^{-a\epsilon ^2/\gamma }h_\lambda ^{n} . \end{aligned}$$

By a similar argument, one can show that

$$\begin{aligned}&\hat{\mathbf {P}}^{\underline{y}}_\lambda ( \underline{x}^\lambda (\gamma ) = \underline{z}, \max _{t\le \gamma } x^\lambda _n(t) \le 2\eta ) \\&\quad \ge \hat{\mathbf {P}}^{\underline{y}}_\lambda ( \underline{x}^\lambda (\gamma ) = \underline{z}) - c_2 \gamma ^{-n/2} e^{-a\eta ^2/\gamma } h_\lambda ^{n} . \end{aligned}$$

Finally, by the standard local limit theorem,

$$\begin{aligned} \hat{\mathbf {P}}^{\underline{y}}_\lambda ( \underline{x}^\lambda (\gamma ) = \underline{z}) \ge c_3 \gamma ^{-n/2} h_\lambda ^{n} . \end{aligned}$$

As a result, we have the bound

$$\begin{aligned} \hat{\mathbf {P}}^{\underline{y}}_{+,\lambda } ( \underline{x}^\lambda (\gamma ) = \underline{z}, \max _{t\le \gamma } x^\lambda _n(t) \le 2\eta ) \ge c_4 h_\lambda ^{n} , \end{aligned}$$

uniformly in \(\underline{y},\underline{z}\in \mathbb {A}_{n, \lambda }^{+, {\mathsf r}}\). Combining this bound with (7.31), we infer that

$$\begin{aligned} \hat{\mathbf {P}}^{\underline{r}}_{1,+,\lambda } ( \underline{x}^\lambda (1) = \underline{z}, \max _{t\le 1} x_n^\lambda (t) \le 2\eta ) \ge c_5 h_\lambda ^{n} , \end{aligned}$$
(7.32)

uniformly in \(\underline{r},\underline{z}\in \mathbb {A}_{n, \lambda }^{+, {\mathsf r}}\). \(\square \)

Appendix 2: Invariance principles for random walks in Weyl chambers

Conditional limit theorems and conditional invariance principles for random walks in different cones have been studied in [6] and [8]. All the results in these papers are proved in the case when the non-rescaled walk starts at a fixed point. In this paragraph, we give certain improvements of these results to the case when the starting point of the non-rescaled walk may grow (but we shall consider walks in Weyl chambers only).

More precisely, we shall consider the following subsets of the Euclidean space:

  • chamber of type A: \(\{x: x_1<x_2<\cdots <x_n\}\);

  • chamber of type C: \(\{x: 0<x_1<x_2<\cdots <x_n\}\);

  • chamber of type D: \(\{x: |x_1|<x_2<\cdots <x_n\}\).

Let \(u_W\) denote the unique (up to a constant multiplier) positive harmonic function on W:

  • if W is the chamber of type A, then \(u_W(x) = \prod _{i<j} (x_j-x_i)\);

  • if W is the chamber of type C, then \(u_W(x) = \prod _k x_k \prod _{i<j} (x^2_j-x^2_i)\);

  • if W is the chamber of type D, then \(u_W(x)=\prod _{i<j} (x^2_j-x^2_i)\).

Proposition 4

Let W be a Weyl chamber of type A, C or D. Let \(\tau \) be the first exit time from W, that is,

$$\begin{aligned} \tau = \inf \{t>0 \,:\, \underline{x}^\lambda (t)\notin W\}. \end{aligned}$$

Then, as \(\underline{r}=\underline{r}_\lambda \rightarrow 0\),

$$\begin{aligned} \hat{\mathbf {P}}^{\underline{r}}_\lambda ( \underline{x}^\lambda (1)\in \,\cdot \;|\, \tau >1 )\rightarrow \mu \quad \text {weakly}, \end{aligned}$$

where \(\mu \) is the probability measure on W with density proportional to \(u_W(x)e^{-|x|^2/2}\).

Furthermore, under \(\hat{\mathbf {P}}^{\underline{r}}_\lambda \), \(\underline{x}^\lambda \) converges weakly on C[0, 1] to the Brownian meander in W started at zero.

By “Brownian meander in W”, we mean a Brownian motion conditioned on staying in W up to time one. If the starting point lies inside W, then one has a condition of positive probability. However, if the starting point lies on the boundary of W, then the probability of the condition is zero and it is not at all clear how one can construct such a process. Garbit [11] has constructed Brownian meanders started at zero for a quite large class of cones. This class includes Weyl chambers.

Proof

The main difference with [6, Theorem 3] is that we find the limit for conditional distributions without determining the asymptotic behavior of \(\hat{\mathbf {P}}^{\underline{r}}_\lambda (\tau >1)\). (Recall once again that [6, Theorem 3] is proven under the assumption \(\underline{r}=h_\lambda a\) for some fixed \(a\in W\).)

Fix some \(\epsilon \in (0,1/2)\) and define the stopping time

$$\begin{aligned} \nu _{\lambda ,\epsilon } = \inf \{t>0 \,:\, \underline{x}^\lambda (t)\in W_{\lambda ,\epsilon }\} , \end{aligned}$$

where

$$\begin{aligned} W_{\lambda ,\epsilon } = \{x\in W \,:\, {\text {dist}}(x,\partial W)\ge H_\lambda ^{-2\epsilon }\} . \end{aligned}$$

According to [6, Lemma 14],

$$\begin{aligned} \hat{\mathbf {P}}^{\underline{r}}_\lambda (\tau> H_\lambda ^{-2\epsilon } , \nu _{\lambda ,\epsilon } > H_\lambda ^{-2\epsilon }) \le e^{-c_1 H_\lambda ^{2\epsilon }} , \end{aligned}$$
(7.33)

uniformly in \(\underline{r}\). Since we consider lattice random walks, there exists \(\underline{r}_0\) such that

$$\begin{aligned} \hat{\mathbf {P}}^{\underline{r}}_\lambda (\tau>1) \ge \hat{\mathbf {P}}^{\underline{r}_0}_\lambda (\tau >1) . \end{aligned}$$

(If W is of type A or C, then we may take \(r_0=h_\lambda (1,2,\ldots ,n)\), while if W is of type D, then we may take \(r_0=h_\lambda (0,1,\ldots ,n-1)\).) According to [6, Theorem 1],

$$\begin{aligned} \hat{\mathbf {P}}^{\underline{r}_0}_\lambda (\tau >1) \sim C_1 h_\lambda ^p , \end{aligned}$$

where p is a positive constant depending on the type of W only. Consequently,

$$\begin{aligned} \hat{\mathbf {P}}^{\underline{r}}_\lambda (\tau >1) \ge C_2 h_\lambda ^p , \end{aligned}$$
(7.34)

uniformly in \(\underline{r}\). Combining (7.33) and (7.34), we infer that

$$\begin{aligned} \frac{ \hat{\mathbf {P}}^{\underline{r}}_\lambda (\tau> H_\lambda ^{-2\epsilon } , \nu _{\lambda ,\epsilon }> H_\lambda ^{-2\epsilon }) }{ \hat{\mathbf {P}}^{\underline{r}}_\lambda (\tau >1) } \rightarrow 0, \quad \lambda \downarrow 0 , \end{aligned}$$
(7.35)

uniformly in \(\underline{r}\). Furthermore, it follows from the exponential Doob inequality that

$$\begin{aligned} \hat{\mathbf {P}}^{\underline{r}}_\lambda \big ( \max _{t\le H_\lambda ^{-2\epsilon }} |\underline{x}^\lambda (t)-\underline{x}^\lambda (0)| > \theta _\lambda \big ) \le e^{-c_2 \theta _\lambda ^2 H_\lambda ^\epsilon } , \end{aligned}$$

where \(\theta _\lambda \rightarrow 0\) sufficiently slowly. This implies that, whenever \(|\underline{r}|\le \theta _\lambda \),

$$\begin{aligned} \frac{ \hat{\mathbf {P}}^{\underline{r}}_\lambda \big ( \max _{t\le H_\lambda ^{-2\epsilon }} |\underline{x}^\lambda (t)|> 2\theta _\lambda \big ) }{ \hat{\mathbf {P}}^{\underline{r}}_\lambda (\tau >1) } \rightarrow 0 . \end{aligned}$$
(7.36)

It follows now from (7.35) and (7.36) that, uniformly in \(\underline{r}\),

$$\begin{aligned} \hat{\mathbf {P}}^{\underline{r}}_\lambda (\tau>1) = (1+o(1)) \, \hat{\mathbf {P}}^{\underline{r}}_\lambda (\tau >1 , \nu _{\lambda ,\epsilon }\le H_\lambda ^{-2\epsilon } , \max _{t\le \nu _{\lambda ,\epsilon }} |\underline{x}(t)| \le 2\theta _\lambda ) \end{aligned}$$
(7.37)

and

$$\begin{aligned}&\hat{\mathbf {P}}^{\underline{r}}_\lambda (\underline{x}^\lambda (1)\in A , \tau>1) \nonumber \\&\quad = (1+o(1))\, \hat{\mathbf {P}}^{\underline{r}}_\lambda (\underline{x}^\lambda (1)\in A , \tau >1 , \nu _{\lambda ,\epsilon }\le H_\lambda ^{-2\epsilon } , \max _{t\le \nu _{\lambda ,\epsilon }} |\underline{x}(t)|\le 2\theta _\lambda )\qquad \end{aligned}$$
(7.38)

for any compact \(A\subset W\).

Using the Markov property at time \(\nu _{\lambda ,\epsilon }\) and applying [6, Lemma 20], we obtain from (7.37) and (7.38)

$$\begin{aligned} \hat{\mathbf {P}}^{\underline{r}}_\lambda (\tau >1) = (c_3+o(1))\, h_\lambda ^p\, \hat{\mathbf {E}}^{\underline{r}}_\lambda \big [u_W(\underline{x}^\lambda (\nu _{\lambda ,\epsilon });\, \nu _{\lambda ,\epsilon }\le H_\lambda ^{-2\epsilon } , \max _{t\le \nu _{\lambda ,\epsilon }} |\underline{x}(t)|\le 2\theta _\lambda \big ] \end{aligned}$$

and

$$\begin{aligned}&\hat{\mathbf {P}}^{\underline{r}}_\lambda (\underline{x}^\lambda (1) \in A , \tau >1) = (c_4+o(1))\, h_\lambda ^p\, \int _A u_W(z) e^{-|z|^2/2} \mathrm{d}z \\&\quad \times \hat{\mathbf {E}}^{\underline{r}}_\lambda \big [ u_W(\underline{x}^\lambda (\nu _{\lambda ,\epsilon }));\, \nu _{\lambda ,\epsilon }\le H_\lambda ^{-2\epsilon } , \max _{t\le \nu _{\lambda ,\epsilon }} |\underline{x}(t)|\le 2\theta _\lambda \big ] . \end{aligned}$$

Thus, the proof of the first statement is completed.

To prove the functional convergence, it suffices to repeat the proof of [8, Theorem 1] using (7.35) and (7.36) instead of the corresponding estimates therein. \(\square \)

Corollary 2

Let W be the chamber of type C. If \(\underline{r}=\underline{r}_\lambda \rightarrow \underline{r}^*\in \partial W\), then the sequence \(\hat{\mathbf {P}}^{\underline{r}}_\lambda (\underline{x}^\lambda (1)\in A \,|\, \tau >1)\) converges weakly. The densities of limiting laws on W has are uniformly bounded. Moreover, \(\underline{x}^\lambda \) converges weakly on C[0, 1] towards the Brownian meander in W started at \(\underline{r}^*\).

Proof

We just split the original set of random walks into a finite number of subsets in such a way that the differences of coordinates of the starting points in every block converge to zero and the differences of coordinates from different blocks stay bounded away from zero. Then, the probability that different blocks do not intersect is bounded away from zero and, consequently, the conditioning on \(\{\tau >1\}\) is equivalent to conditioning every block on staying in the corresponding chamber. (If \(r^*_1>0\), then every block is a random block in a chamber of type A, while if \(r^*_1=0\), then the lowest block is a random walk in a chamber of type C and all other blocks are random walks in chambers of type A.) \(\square \)

Proof (Proof of (I.2))

[Proof of (I.2)] Assume that there exists a sequence \(\underline{r}(j)\) such that

$$\begin{aligned} \hat{\mathbf {P}}^{\underline{r}(j)}_{1,\lambda } \big ( \max _{t\le 1} x^\lambda _n(t)\le 2\eta \bigm | \tau >1 \big ) \rightarrow 0 . \end{aligned}$$

Since we are looking at starting points \(\underline{r}\) with \(r_n\le \eta \), there exists a convergent subsequence \(\underline{r}(j_k)\). Let \(\underline{r}^*\) denote the limiting point. It follows immediately from the usual functional CLT that the case \(\underline{r}^*\in W\) is impossible. But, if \(\underline{r}^*\in \partial W\), then we may use Corollary 2 to conclude that the Brownian meander in W started at \(\underline{r}^*\) leaves the set \(\{\underline{x}\in W \,:\, x_n\le 2\eta \}\) with probability one. However, this would contradict [18, Theorem 3.2]. Thus,

$$\begin{aligned} \inf _{\underline{r}\,:\, r_n\le \eta } \hat{\mathbf {P}}^{\underline{r}}_{1,\lambda } \big ( \max _{t\le 1} x^\lambda _n(t)\le 2\eta \bigm | \tau>1 \big ) > 0 , \end{aligned}$$
(7.39)

which implies (I.2). \(\square \)

Proof (Proof of (I.3))

[Proof of (I.3)] Fix some \(\epsilon >0\) and define

$$\begin{aligned} W_{\le \epsilon } = \{\underline{x}\in W \,:\, |x_{i+1}-x_i|\le \epsilon \text { for some } i\ge 0 \} . \end{aligned}$$

Assume that there exists a sequence \(\underline{r}(j)\) such that

$$\begin{aligned} \hat{\mathbf {P}}^{\underline{r}(j)}_{1,\lambda } \big (\underline{x}^\lambda (1)\in W_{\le \epsilon } \,;\, \max _{t\le 1} x_n^\lambda (t)\le 2\eta \bigm | \tau >1 \big ) \ge \epsilon ^{1/2} . \end{aligned}$$

We may again assume that \(\underline{r}(j)\) converges to \(\underline{r}^*\) and this limiting point can not lie in W. But, if \(\underline{r}^*\) is on the boundary of W, then the conditions of Corollary 2 are satisfied and the contradiction follows now from the boundedness of the density of the limiting law and the fact that \(vol(W_{\le \epsilon } \cap \{\underline{x}\,:\, x_n\le 2\eta \}) \le C_3 \eta ^{n-1}\epsilon \).

As a consequence we have that, for all \(\epsilon \) small enough,

$$\begin{aligned} \inf _{\underline{r}\,:\, r_n\le \eta } \hat{\mathbf {P}}^{\underline{r}}_{1,+,\lambda } \big (\underline{x}^\lambda (1)\in \mathbb {A}_{n, \lambda }^{+, {\mathsf r}}, \max _{t\le 1} x^\lambda _n(t)\le 2\eta \bigm | \tau >1 \big ) \ge 1- \epsilon ^{1/2} . \end{aligned}$$
(7.40)

Combining (7.39) and (7.40), we conclude that (I.3) holds for \(t=1\) and all \(\epsilon \) sufficiently small. Using Brownian scaling, we conclude that (I.3) is valid for all \(t>0\). \(\square \)

Rights and permissions

Reprints and permissions

About this article

Check for updates. Verify currency and authenticity via CrossMark

Cite this article

Ioffe, D., Velenik, Y. & Wachtel, V. Dyson Ferrari–Spohn diffusions and ordered walks under area tilts. Probab. Theory Relat. Fields 170, 11–47 (2018). https://doi.org/10.1007/s00440-016-0751-z

Download citation

  • Received:

  • Revised:

  • Published:

  • Issue Date:

  • DOI: https://doi.org/10.1007/s00440-016-0751-z

Keywords

Mathematics Subject Classification

Navigation